Last visit was: 26 Apr 2024, 14:42 It is currently 26 Apr 2024, 14:42

Close
GMAT Club Daily Prep
Thank you for using the timer - this advanced tool can estimate your performance and suggest more practice questions. We have subscribed you to Daily Prep Questions via email.

Customized
for You

we will pick new questions that match your level based on your Timer History

Track
Your Progress

every week, we’ll send you an estimated GMAT score based on your performance

Practice
Pays

we will pick new questions that match your level based on your Timer History
Not interested in getting valuable practice questions and articles delivered to your email? No problem, unsubscribe here.
Close
Request Expert Reply
Confirm Cancel
SORT BY:
Date
Tags:
Show Tags
Hide Tags
Intern
Intern
Joined: 18 May 2023
Posts: 28
Own Kudos [?]: 10 [0]
Given Kudos: 15
Location: India
Concentration: General Management, Leadership
GMAT 1: 600 Q48 V25
Send PM
Math Expert
Joined: 02 Sep 2009
Posts: 92948
Own Kudos [?]: 619232 [1]
Given Kudos: 81609
Send PM
Manager
Manager
Joined: 17 Sep 2023
Posts: 130
Own Kudos [?]: 54 [0]
Given Kudos: 476
Send PM
Math Expert
Joined: 02 Sep 2009
Posts: 92948
Own Kudos [?]: 619232 [1]
Given Kudos: 81609
Send PM
Re: NEW!!! Tough and tricky exponents and roots questions [#permalink]
1
Kudos
Expert Reply
ruis wrote:
In question number 1, once I reach: 50 + 2 * (sqr root of (25^2 - 10 * sqr root of 6))^2 could I operate in the following way?
Pass the 25 to the left side and multiply by 2 so: 50 + 50 * (sqr root of -600)?
Then do decompose 600 and take the square out of the square root so that: 100*2*5 * (sqr root of -3)
It does not work as I do not get to the right answer... Can anyone please help me understand what I am doing wrong here?


When you factor out a term from an expression, ensure to factor that term out from every part of the expression. For example, in ax + bx, factoring out x means taking x out of both ax and bx, which gives x(a + b). Thus, you cannot just factor out 25^2 from only the first term in \(\sqrt{25^2-(10\sqrt{6})^2)}\).
Manager
Manager
Joined: 17 Sep 2023
Posts: 130
Own Kudos [?]: 54 [0]
Given Kudos: 476
Send PM
Re: NEW!!! Tough and tricky exponents and roots questions [#permalink]
Bunuel wrote:
6. If \(x=\sqrt[5]{-37}\) then which of the following must be true?
A. \(\sqrt{-x}>2\)
B. x>-2
C. x^2<4
D. x^3<-8
E. x^4>32

MUST KNOW FOR THE GMAT:

• Even roots from a negative number are undefined on the GMAT (as GMAT is dealing only with Real Numbers): \(\sqrt[{even}]{negative}=undefined\), for example, \(\sqrt{-25}=undefined\).

• Odd roots have the same sign as the base of the root. For example, \(\sqrt[3]{125} =5\) and \(\sqrt[3]{-64} =-4\).

BACK TO THE ORIGINAL QUESTION:

As \(-2^5=-32\), then \(x\) must be a little bit less than -2, hence \(x=\sqrt[5]{-37} \approx -2.1 \lt -2\). Thus \(x^3 \approx (-2.1)^3 \approx -8.something \lt -8\), so option D must be true.

As for the other options:

A. \(\sqrt{-x}=\sqrt{-(-2.1)}=\sqrt{2.1} \lt 2\), \(\sqrt{-x} \gt 2\) is not true.

B. \(x \approx -2.1 \lt -2\), thus \(x \gt -2\) is also not true.

C. \(x^2 \approx (-2.1)^2=4.something \gt 4\), thus \(x^2 \lt 4\) is also not true.

Answer: D.



How about answer choice E? E looks true as well as D
Manager
Manager
Joined: 17 Sep 2023
Posts: 130
Own Kudos [?]: 54 [0]
Given Kudos: 476
Send PM
NEW!!! Tough and tricky exponents and roots questions [#permalink]
Bunuel wrote:
7. If \(x=\sqrt{10}+\sqrt[3]{9}+\sqrt[4]{8}+\sqrt[5]{7}+\sqrt[6]{6}+\sqrt[7]{5}+\sqrt[8]{4}+\sqrt[9]{3}+\sqrt[10]{2}\), then which of the following must be true?

A. \(x \gt 12\)
B. \(10 \lt x \lt 12\)
C. \(8 \lt x \lt 10\)
D. \(6 \lt x \lt 8\)
E. \(x \lt 6\)


Here is a useful little trick: any positive integer root of a number greater than 1 will be more than 1. For example: \(\sqrt[1000]{2} \gt 1\).

Now, \(\sqrt{10} \gt 3\) (as \(3^2=9\)) and \(\sqrt[3]{9} \gt 2\) (as \(2^3=8\)).

Therefore:

    \(x=\sqrt{10}+\sqrt[3]{9}+\sqrt[4]{8}+\sqrt[5]{7}+\sqrt[6]{6}+\sqrt[7]{5}+\sqrt[8]{4}+\sqrt[9]{3}+\sqrt[10]{2}=\)

    \(=(\text{number more than 3})+(\text{number more than 2})+(\text{7 numbers more than 1})=\)

    \(=(\text{number more than 5})+(\text{number more than 7})=\)

    \(=(\text{number more than 12})\)


Answer: A
Answer: E.


Is there another way to do this exercise? I tried calculating the maximun and the minimum to understand the range of x. We have the sum of 9 terms. If all terms were equal to the largest term (sqrt{10}), we would have: sum = 9* (sqrt{10}) with is aprox 27 (a bit more than 27). So, the actual sum must be less than 27.
Then, we do the same with the smallest: sum = 9* 2^(1/10).... 3^2*2^(1/10). I struggle solving that last expression...
Math Expert
Joined: 02 Sep 2009
Posts: 92948
Own Kudos [?]: 619232 [0]
Given Kudos: 81609
Send PM
Re: NEW!!! Tough and tricky exponents and roots questions [#permalink]
Expert Reply
ruis wrote:
Bunuel wrote:
6. If \(x=\sqrt[5]{-37}\) then which of the following must be true?
A. \(\sqrt{-x}>2\)
B. x>-2
C. x^2<4
D. x^3<-8
E. x^4>32

MUST KNOW FOR THE GMAT:

• Even roots from a negative number are undefined on the GMAT (as GMAT is dealing only with Real Numbers): \(\sqrt[{even}]{negative}=undefined\), for example, \(\sqrt{-25}=undefined\).

• Odd roots have the same sign as the base of the root. For example, \(\sqrt[3]{125} =5\) and \(\sqrt[3]{-64} =-4\).

BACK TO THE ORIGINAL QUESTION:

As \(-2^5=-32\), then \(x\) must be a little bit less than -2, hence \(x=\sqrt[5]{-37} \approx -2.1 \lt -2\). Thus \(x^3 \approx (-2.1)^3 \approx -8.something \lt -8\), so option D must be true.

As for the other options:

A. \(\sqrt{-x}=\sqrt{-(-2.1)}=\sqrt{2.1} \lt 2\), \(\sqrt{-x} \gt 2\) is not true.

B. \(x \approx -2.1 \lt -2\), thus \(x \gt -2\) is also not true.

C. \(x^2 \approx (-2.1)^2=4.something \gt 4\), thus \(x^2 \lt 4\) is also not true.

Answer: D.



How about answer choice E? E looks true as well as D


\(x=\sqrt[5]{-37} \approx -2.1 \lt -2\).

2^4 = 16, so (-2.1)^4 will be a little greater than 15 bur for sure less than 32.
Math Expert
Joined: 02 Sep 2009
Posts: 92948
Own Kudos [?]: 619232 [0]
Given Kudos: 81609
Send PM
Re: NEW!!! Tough and tricky exponents and roots questions [#permalink]
Expert Reply
ruis wrote:
Bunuel wrote:
7. If \(x=\sqrt{10}+\sqrt[3]{9}+\sqrt[4]{8}+\sqrt[5]{7}+\sqrt[6]{6}+\sqrt[7]{5}+\sqrt[8]{4}+\sqrt[9]{3}+\sqrt[10]{2}\), then which of the following must be true?

A. \(x \gt 12\)
B. \(10 \lt x \lt 12\)
C. \(8 \lt x \lt 10\)
D. \(6 \lt x \lt 8\)
E. \(x \lt 6\)


Here is a useful little trick: any positive integer root of a number greater than 1 will be more than 1. For example: \(\sqrt[1000]{2} \gt 1\).

Now, \(\sqrt{10} \gt 3\) (as \(3^2=9\)) and \(\sqrt[3]{9} \gt 2\) (as \(2^3=8\)).

Therefore:

    \(x=\sqrt{10}+\sqrt[3]{9}+\sqrt[4]{8}+\sqrt[5]{7}+\sqrt[6]{6}+\sqrt[7]{5}+\sqrt[8]{4}+\sqrt[9]{3}+\sqrt[10]{2}=\)

    \(=(\text{number more than 3})+(\text{number more than 2})+(\text{7 numbers more than 1})=\)

    \(=(\text{number more than 5})+(\text{number more than 7})=\)

    \(=(\text{number more than 12})\)


Answer: A
Answer: E.


Is there another way to do this exercise? I tried calculating the maximun and the minimum to understand the range of x. We have the sum of 9 terms. If all terms were equal to the largest term (sqrt{10}), we would have: sum = 9* (sqrt{10}) with is aprox 27 (a bit more than 27). So, the actual sum must be less than 27.
Then, we do the same with the smallest: sum = 9* 2^(1/10).... 3^2*2^(1/10). I struggle solving that last expression...


The approximation is pretty much the only way. However, you can check slight variations here: https://gmatclub.com/forum/m09-183829.html
GMAT Club Bot
Re: NEW!!! Tough and tricky exponents and roots questions [#permalink]
   1   2   3   4 
Moderators:
Math Expert
92948 posts
Senior Moderator - Masters Forum
3137 posts

Powered by phpBB © phpBB Group | Emoji artwork provided by EmojiOne